QR factorization of a n x 1 matrix

Click For Summary
The discussion focuses on the QR factorization of an n x 1 matrix, specifically addressing the reduced QR factorization of a vector a. The user outlines the Gram-Schmidt process, identifying the first vector as u1 = a1 and normalizing it to find e1. The resulting matrices are Q, which is a normalized version of a1, and R, a 1x1 matrix representing the norm of a1. The user expresses confusion about the simplicity of the case and its implications for understanding the concept. Overall, the thread highlights the challenges of applying QR factorization to a single-column matrix while preparing for an upcoming exam.
abajaj2280
Messages
2
Reaction score
0

Homework Statement


Consider the vector a as an n × 1 matrix.

A) Write out its reduced QR factorization, showing the matrices \hat{Q} and \hat{R} explicitly.

B) What is the solution to the linear least squares problem ax ≃ b where b is a given n-vector.


Homework Equations


I was using the equation from 1.1 (http://www.math.ucla.edu/~yanovsky/Teaching/Math151B/handouts/GramSchmidt.pdf) to help me solve this problem.

The Attempt at a Solution


I haven't taken linear algebra for about 2 years and this is kind of hazy. I'm really confused here, and I really don't know where to start. I know that I'm supposed to come to this website with some sort of progress, but I'm really confused here.
 
Physics news on Phys.org
So, the first step in the Gram-Schmidt process is to think of the matrix as being a row of column vectors. Since your matrix is n x 1, it's like having a matrix of only one column vector. Thus A = [\mathbf{a_1}] in this case. So, going by the pdf you provided, let \mathbf{u_1} = \mathbf{a_1} and then
\mathbf{e_1} = \frac{\mathbf{u_1}}{\left\| \mathbf{u_1} \right\|} = \frac{\mathbf{a_1}}{\sqrt{(a_{11})^2+(a_{21})^2+...+(a_{n1}^2)}}.

In this case, Q = [\frac{ \mathbf{a_1} }{\left\| \mathbf{a_1} \right\|}] and R is the 1x1 matrix
[ \mathbf{a_1} \bullet (\frac{ \mathbf{a_1} }{\left\| \mathbf{a_1} \right\|}) ] = [ \frac{ \mathbf{a_1} \bullet \mathbf{a_1} }{ \left\| \mathbf{a_1} \right\| } ] = [\frac{(\left\| \mathbf{a_1} \right\|)^2}{\left\| \mathbf{a_1} \right\|}] = [\left\|\mathbf{a_1}\right\|].

Then, QR = [ \frac{\mathbf{a_1}}{\left\|\mathbf{a_1}\right\|} \left\|\mathbf{a_1}\right\| ] = [\mathbf{a_1}].

In this case, the result isn't very interesting, because it's an nx1 matrix. But I think that's also to (supposedly) make it easier for you. I can see how in this case it made it even more confusing.
 
Thank you so much for your help. I have an exam in this class in one week, so I will be referring back to this when studying.
 
Question: A clock's minute hand has length 4 and its hour hand has length 3. What is the distance between the tips at the moment when it is increasing most rapidly?(Putnam Exam Question) Answer: Making assumption that both the hands moves at constant angular velocities, the answer is ## \sqrt{7} .## But don't you think this assumption is somewhat doubtful and wrong?

Similar threads

  • · Replies 4 ·
Replies
4
Views
2K
  • · Replies 2 ·
Replies
2
Views
3K
  • · Replies 1 ·
Replies
1
Views
4K
Replies
15
Views
2K
  • · Replies 10 ·
Replies
10
Views
2K
  • · Replies 3 ·
Replies
3
Views
2K
  • · Replies 1 ·
Replies
1
Views
2K
  • · Replies 15 ·
Replies
15
Views
3K
  • · Replies 9 ·
Replies
9
Views
6K
  • · Replies 4 ·
Replies
4
Views
2K